Conteúdo

Cálculo Diferencial e Integral I

Exercícios

Números reais

  1. Simplifique as seguintes expressões (definidas nos respectivos domínios):

    1. $\displaystyle \frac{\frac{x}{2}}{\frac{2}{x}}$,
      Sugestão

      Note o domínio da expressão: $\mathbb{R}\setminus\{0\}$.

      Solução

      $\displaystyle \frac{\frac{x}{2}}{\frac{2}{x}}=\frac{x^2}{4}$, se $x\neq 0$.

    2. $\frac{x+1}{\frac{1}{x}+1}$,
      Sugestão

      Note o domínio da expressão...

      Solução

      $\frac{x+1}{\frac{1}{x}+1}=x$, se $x\neq 0$ e $x\neq -1$.

    3. $\frac{1}{1+x} + \frac{1}{x^2+x}$,
      Sugestão

      Note o domínio da expressão...

      Solução

      $\frac{1}{1+x} + \frac{1}{x^2+x}=\frac{1}{x}$, se $x\neq -1$ (e $x\neq 0$).

    4. $\sqrt{x^2}$,
      Sugestão

      $x^2=(-x)^2=|x|^2$.

      Solução

      $\sqrt{x^2}=|x|=\begin{cases}x, \text{se } x\geq 0, \\ -x, \text{se } x\leq 0,\end{cases}$ para todo o $x\in\mathbb{R}$.

    5. $\left(\sqrt{x}\right) ^2$,
      Solução

      $\left(\sqrt{x}\right) ^2=x$, para $x\geq 0$.

    6. $4^x \frac{4}{2^x}$,
      Solução

      $2^{x+2}$

    7. $2^{x^2}\left( 2^x\right) ^2$,
      Solução

      $2^{x(x+2)}$

    8. $\frac{\sqrt[3]{x^2}}{\sqrt[6]{x}}$,
      Sugestão

      Use potências com expoente fraccionário.

      Solução

      $\sqrt{x}$, para $x\gt 0$.

    9. $\sqrt{x-2}\sqrt{x+2}$,
      Solução

      $\sqrt{x^2-4}$, para $x\geq 2$.

    10. $\frac{\sqrt{x}}{\sqrt{x+1}-\sqrt{x}}$,
      Solução

      $\sqrt{x(x+1)}+x$, para $x\geq 0$.

    11. $\log\left( \frac{1}{x}\right) +\log\left( x^{2}\right) $,
      Sugestão

      Relembre as propriedades da função logaritmo. Para $a,b\gt 0$, $\log(ab)=\log a + \log b$, $\log(a/b)=\log a - \log b$, $\log 1 = 0$.

      Solução

      $\log(x)$, para $x\gt 0$.

    12. $\log\left( 2x^2+2x^{-2}\right) +\log\left( \frac{x^2}{2}+\frac{x^{-2}}{2}\right)$.
      Solução

      $2\log(x^2+x^{-2}) $ ou $2\log(x^4+1)-4\log(|x|) $, para $x\neq 0$.

  2. Resolva as seguintes equações e inequações:

    1. $(x^{2}-3x+2)(x-1)\geq 0$,
      Solução

      $x=1$ ou $x\geq 2$.

    2. $x\leq 2 - x^{2} $,
      Solução

      $ -2 \leq x\leq 1$.

    3. $x^{2}\leq 2-x^{4}$,
      Sugestão

      Comece por usar o resultado da alínea anterior.

      Solução

      $-1\leq x\leq 1$

    4. $x^3+x\leq 2x^2$,
      Solução

      $x\leq 0$ ou $x=1$.

    5. $\sqrt[3]{x^2+2x}=2$,
      Solução

      $x=-4$ ou $x=2$.

    6. $\sqrt[3]{x-1}=\sqrt{x-1}$,
      Sugestão

      Comece por pôr-se a questão: quando é que uma raíz cúbica é igual a uma raíz quadrada?.

      Solução

      $x=1$ ou $x=2$.

    7. $\frac{x-1}{x^{2}-1}\leq 1$,
      Solução

      $x\lt -1$ ou $0\leq x\lt 1$ ou $x\gt 1$.

    8. $x=\frac{1}{x}$,
      Solução

      $x=1$ ou $x= -1$

    9. $x\lt \frac{1}{x}$,
      Solução

      $0\lt x \lt 1$ ou $x\lt -1$.

    10. $x\lt |x|$,
      Solução

      $x\lt 0$

    11. $|x|\geq \frac{x}{2}+1$,
      Solução

      $x\geq 2$ ou $x\leq - \frac{2}{3}$.

    12. $|x|\leq |x-2|$,
      Sugestão

      Quando é que a distância de $x$ a $0$ é menor ou igual à distância de $x$ a $2$?

      Solução
      A solução corresponde aos valores de $x$ para os quais o gráfico de $|x|$ está "abaixo" de $|x-2|$, i.e., $x\leq 1$.
    13. $|x^2-2|\leq 2$,
      Solução

      $-2\leq x\leq 2$.

    14. $\frac{x^4-16}{|x-1|}\leq 0$,
      Solução

      $-2\leq x\lt 1$ ou $1\lt x\leq 2$.

    15. $e^{x^3}\lt 1$,
      Solução

      $x\lt 0$.

    16. $e^{-2x}-2e^{-x}\leq -1$,
      Sugestão

      $y=e^{-x}$.

      Solução

      $x=0$.

    17. $\log\left( \frac{1}{x}\right) \geq 0$,
      Solução

      $0\lt x\leq 1$.

    18. $\log(x^2-3)\geq 0$,
      Solução

      $x\leq -2$ ou $x\geq 2$.

    19. $\sen(1/x)=0$,
      Solução

      $x=1/k\pi$ com $k\in\mathbb{Z}\setminus \{0\}$.

    20. $\sen x \gt \cos x$.
      Solução

      $x\in \cup_{k\in \mathbb{Z}}{\left]\frac{\pi}{4}+2k\pi, \frac{\pi}{4}+(2k+1)\pi\right[}$.

  3. Escreva cada um dos seguintes conjuntos como intervalos ou reuniões de intervalos:

    1. $\left\{x:\frac{x-1}{x+1}\leq 1\right\}$,
      Solução

      $]-1, +\infty[$.

    2. $\left\{x:\frac{x^{4}-1}{x^{3}}\leq x\right\}$,
      Solução

      $\left]0, +\infty\right[$.

    3. $\{x: |3x-4|\geq x^{2}\}$,
      Solução

      $\left[-4, 1\right]$.

    4. $\{x:|x-1|(x^{2}-4)\geq 0\}$,
      Solução

      $\left]-\infty,-2\right]\cup \{1\} \cup\left[2,+\infty\right[$.

    5. $\{x:(|x|-1)(x^{2}-4)\leq 0\}$,
      Solução

      $\left[-2, -1\right]\cup\left[1, 2\right] $.

    6. $\{x:|x^2-1|\leq |x+1|\}$,
      Solução

      $\{-1\}\cup \left[ 0,2\right] $.

    7. $\{x:x^{2}-|x|-2\leq 0\}$,
      Solução

      $\left[ -2,2\right] $.

    8. $\left\{x: \frac{x}{|x|-1}\geq 0\right\}$,
      Solução

      $\left]-1, 0\right]\cup\left]1, +\infty\right]$.

    9. $\left\{ x: \frac{x^2-|x|}{x-3}\leq 0\right\}$,
      Solução

      $\left]-\infty,-1\right]\cup\left\{0\right\}\cup\left[1, 3\right[$.

    10. $\left\{ x: \tg x \gt \sqrt{3} \right\}$.
      Solução

      $\displaystyle\bigcup_{k\in\mathbb{Z}}{\left]\frac{\pi}{3}+k\pi,\frac{\pi}{2}+k\pi\right[}$.

  4. Indique justificando quais das proposições seguintes são verdadeiras:

    1. $\{1\}\subset\{1,\{2,3\}\}$
      Solução

      Verdadeira.

    2. $\{1\}\in\{1,\{2,3\}\}$
      Solução

      Falsa.

    3. $2\in\{1,\{2,3\}\}$
      Solução

      Falsa.

    4. $1\in\{\mathbb{R}\}$
      Solução

      Falsa.

    5. $\emptyset=\{x\in\mathbb{N}:x=x+1\}$
      Solução

      Verdadeira.

    6. $\emptyset \in\{0\}$
      Solução

      Falsa.

    7. $\emptyset \subset\{0\}$
      Sugestão

      Se a proposição fosse falsa existiria $x\in\emptyset$...

      Solução

      Verdadeira.

    8. $\forall_{x\in\mathbb{R}}\, x\gt 0 \; \Leftrightarrow \; x^{-1}\gt 0$
      Solução

      Verdadeira.

    9. $\forall_{x\in\mathbb{R}}\, x\gt 1 \;\Leftrightarrow \; x^{-1} \lt 1$
      Solução

      Falsa.

    10. $\forall_{x,y\in\mathbb{R}}\, x\lt y \;\Rightarrow y^{-1} \lt x^{-1}$
      Solução

      Falsa.

    11. $\forall_{x\neq 0}\,x^2\gt 0$
      Solução

      Verdadeira.

    12. $\forall_{x,y\in\mathbb{R}}\, x\lt y \;\Rightarrow x^2 \lt y^2$
      Solução

      Falsa.

    13. $\forall_{x,y\in\mathbb{R}}\, x\lt y\lt 0 \;\Rightarrow x^2 \gt y^2$
      Solução

      Verdadeira.

  5.  Inserir/Editar âncora(Ex. 1.17, 1.18 e 1.19 de [2]) Demonstre pelo princípio de indução matemática que:

    1. $\sum^{n}_{k=1}(2k-1)=1+3+\dots+(2n-1)=n^{2},\; \forall n\in\mathbb{N}_{1}$.
      Comentário

      A primeira igualdade é simplesmente um relembrar da notação para somas envolvendo somatórios. Esta notação, embora mais compacta e precisa, pode ser menos intuitiva quando se encontra pela primeira vez.

      Solução

      Para $n=1$, temos $2\cdot 1 -1 =1$, que é uma proposição verdadeira.

      Hipótese de indução: para um certo $n\in \mathbb{N}_{1}$, temos $1+3+\dots+(2n-1)=n^{2}$.

      Tese (a provar): $1+3+\dots+(2n-1)+(2(n+1)-1)=(n+1)^{2}$.

      Demonstração: Usando a hipótese de indução (na primeira igualdade), temos: \[1+3+\dots+(2n-1)+(2(n+1)-1)=n^2+(2n+2-1)=n^2+2n+1=(n+1)^2\] como queríamos mostrar.

    2. $\frac{1}{1\cdot 2}+\frac{1}{2\cdot3}+\ldots+\frac{1}{n(n+1)}=\frac{n}{n+1}$, para todo o natural $n\geq 1$.
      Solução

      Para $n=1$, temos $\frac{1}{1\cdot 2}=\frac{1}{1+1}\Leftrightarrow \frac{1}{2}=\frac{1}{2}$, que é uma proposição verdadeira.

      Hipótese de indução: para um certo $n\in \mathbb{N}_{1}$, temos $\frac{1}{1\cdot 2}+\frac{1}{2\cdot 3}+\ldots+\frac{1}{n(n+1)}=\frac{n}{n+1}$.

      Tese (a provar): $\frac{1}{1.2}+\frac{1}{2.3}+\ldots+\frac{1}{n(n+1)}+\frac{1}{(n+1)(n+2)}=\frac{n+1}{n+2}$.

      Demonstração: Usando a hipótese de indução, temos: \begin{align*} \frac{1}{1\cdot 2}+\frac{1}{2\cdot 3}+\ldots+\frac{1}{n(n+1)}+\frac{1}{(n+1)(n+2)} & = \frac{n}{n+1}+\frac{1}{(n+1)(n+2)}\\ & = \frac{n(n+2)+1}{(n+1)(n+2)}=\frac{n^2+2n+1}{(n+1)(n+2)}\\ & = \frac{(n+1)^2}{(n+1)(n+2)}=\frac{n+1}{n+2} \end{align*} como queríamos mostrar.

    3. $(n!)^2 \gt 2^nn^2$, para todo o natural $n\geq 4$.
      Solução

      Para $n=4$, temos $(4!)^2 \gt 2^4 4^2 \Leftrightarrow 4^2 3^2 2^2 \gt 4^2 2^2 2^2 \Leftrightarrow 3^2 \gt 2^2 \Leftrightarrow 9 \gt 4$, que é uma proposição verdadeira.

      Hipótese de indução: para um certo $n\geq 4$, temos $(n!)^2 \gt 2^nn^2$.

      Tese: $((n+1)!)^2 \gt 2^{n+1}(n+1)^2$.

      Demonstração: Usando a hipótese de indução (no segundo passo) temos: $((n+1)!)^2 = (n+1)^2 (n!)^2 \gt (n+1)^2 2^n n^2$.

      Comparando com o minorante que pretendemos obter, $2^{n+1}(n+1)^2$, verificamos que basta mostrar que $n^2\geq 2$. Tal é verdade pois, sendo $n\geq 4$, temos $n^2 \geq 16 \gt 2$.

    4. $n!\geq 2^{n-1}$, para todo o natural $n\geq 1$.
  6.  Inserir/Editar âncoraDemonstre pelo princípio de indução matemática que:

    1. $1+2+3+\dots+n=\frac{n(n+1)}{2}$, para qualquer $n\in\mathbb{N}_{1}$.
    2. Para $a\in\mathbb{R}$, $(a-1)(1+a+\dots +a^n)=a^{n+1}-1,$ para qualquer $n\in\mathbb{N}$.
    3. $\frac{1}{2!}+\frac{2}{3!}+\dots +\frac{n}{(n+1)!}=1-\frac{1}{(n+1)!}$, para qualquer $n\in\mathbb{N}_1$.
      Solução

      Para $n=1$, obtemos $\frac{1}{2!}=1-\frac{1}{(1+1)!}$, que é uma proposição verdadeira.

      Hipótese de indução: para um certo $n\in\mathbb{N}_1$, $\frac{1}{2!}+\frac{2}{3!}+\dots +\frac{n}{(n+1)!}=1-\frac{1}{(n+1)!}$.

      Tese: $\frac{1}{2!}+\frac{2}{3!}+\dots +\frac{n}{(n+1)!}+\frac{n+1}{(n+2)!}=1-\frac{1}{(n+2)!}.$

      Demonstração: Usando a hipótese de indução, \begin{align*} \frac{1}{2!}+\frac{2}{3!}+\dots +\frac{n}{(n+1)!}+\frac{n+1}{(n+2)!} &= \left( 1-\frac{1}{(n+1)!}\right) +\left( \frac{n+1}{(n+2)!}\right) \\ & = 1- \frac{n+2-n-1}{(n+2)!}\\ & = 1- \frac{1}{(n+2)!} \end{align*} como queríamos mostrar.

    4. $1^{3}+2^{3}+3^{3}+\dots+n^{3}=\left( \frac{n(n+1)}{2}\right)^{2}$, para qualquer $n\in\mathbb{N}_{1}$.
    5. $1+\frac{1}{\sqrt{2}}+\dots + \frac{1}{\sqrt{n}}\geq \sqrt{n}$, para qualquer $n\in\mathbb{N}_{1}$.
      Solução

      Para $n=1$ obtemos $1\geq \sqrt{1}$ que é uma proposição verdadeira.

      Hipótese de indução: para um certo $n\in\mathbb{N}_1$, $1+\frac{1}{\sqrt{2}}+\dots + \frac{1}{\sqrt{n}}\geq \sqrt{n}$.

      Tese: $1+\frac{1}{\sqrt{2}}+\dots + \frac{1}{\sqrt{n}}+\frac{1}{\sqrt{n+1}}\geq \sqrt{n+1}$.

      Demonstração: Usando a hipótese de indução $1+\frac{1}{\sqrt{2}}+\dots + \frac{1}{\sqrt{n}}+\frac{1}{\sqrt{n+1}}\geq \sqrt{n}+\frac{1}{\sqrt{n+1}}=\frac{\sqrt{n}\sqrt{n+1}+1}{\sqrt{n+1}}$.

      A demonstração ficará concluída se provarmos que esta última expressão é maior ou igual a $\sqrt{n+1}$. Convém para tal dar um aspecto que facilite a comparação a $\sqrt{n+1}$. Como \[\sqrt{n+1}=\frac{n+1}{\sqrt{n+1}}\] reduzimos o fazer a comparação a minorar $\sqrt{n}\sqrt{n+1}$ por $n$. Tal de facto é possível pois $\sqrt{n}\sqrt{n+1}=\sqrt{n^2+n}\geq \sqrt{n^2} = n$.

  7.  Inserir/Editar âncoraDemonstre pelo princípio de indução matemática que:

    1. $(n+2)!\geq 2^{2n}$, para qualquer $n\in\mathbb{N}_1$.
    2. $2n-3 \lt 2^{n-2}$, para todo o natural $n\geq 5$.
    3. $7^{n}-1$ é múltiplo de $6$ para qualquer $n\in\mathbb{N}_{1}$.
      Solução

      Para $n=1$, temos $7^1-1=6$, que é divisível por $6$.

      Hipótese de indução: para um certo $n\in\mathbb{N}_{1}$, $7^{n}-1$ é divisível por $6$. Isto significa que existe $k\in\mathbb{N}_1$ tal que $6k=7^n-1$.

      Tese: $7^{n+1}-1$ é divisível por $6$, isto é, existe um natural positivo $j$ tal que $7^{n+1}-1=6j$.

      Demonstração: \[ 7^{n+1}-1=7\cdot 7^n-1= 7 (7^n-1+1)-1=7(6k+1)-1= 6\cdot 7 k + 7 -1=6(7 k+1) \] em que na terceira igualdade usámos a hipótese de indução. Demonstrámos então a tese com $j=7 k+1$.

    4. $2^{2n}+2$ é múltiplo de $3$ para qualquer $n\in\mathbb{N}$.
  8. (Ex. 1.20 de [2]) Demonstre a desigualdade de Bernoulli:

    Sendo $a\gt -1$ e $n\in\mathbb{N}$, $(1+a)^n\geq 1+na$.
  9. Seja $P(n)$ a condição “$n^2+3n+1$ é par”.

    1. Mostre que $P(n)\Rightarrow P(n+1)$.
    2. Pode concluir que $n^2+3n+1$ é par, para qualquer $n\in\mathbb{N}$?
    3. Mostre que, para qualquer $n\in\mathbb{N}$, $n^2+3n+1$ é ímpar.
    Solução
    1. Vamos ver que $P(n)\Rightarrow P(n+1)$, ou seja, que se $n^2+3n+1$ é par, também $(n+1)^2+3(n+1)+1$ é par. Temos \[(n+1)^2+3(n+1)+1=n^2+2n+1+3n+3+1=(n^2+3n+1)+2n+4.\] Assumindo que $n^2+3n+1$ é par, como $2n+4=2(n+2)$ é também par, conclui-se que $(n+1)^2+3(n+1)+1$ sendo uma soma de números pares será par.
    2. Não.
    3. Indução... (Como acima: se $n^2+3n+1$ é ímpar, $(n+1)^2+3(n+1)+1$ será uma soma de um número ímpar com um número par, e será portanto ímpar. Mas neste caso $P(0)$ é verdadeira: $1$ é ímpar.)
  10. Seja $f: \mathbb{N}\longrightarrow \mathbb{N}$ tal que $f(0)=1$ e $f(n+1)=(2n+2)(2n+1) f(n)$. Mostre por indução matemática que, para qualquer $n\in \mathbb{N}$,

    \[f(n)=\left( 2n\right)!\]
  11. Considere a sucessão real $(u_n)$ dada por:

    \[ \begin{cases} u_1=3,\\ u_{n+1} =\frac{3u_n}{(n+1)^2}. \end{cases} \]

    Mostre, usando indução matemática, que $u_n=\frac{3^n}{(n!)^2}$, para qualquer $n\in\mathbb{N}_{1}$.

  12. (Teste de 29/4/2006) Considere a sucessão real $(u_n)$ dada por:

    \[ \begin{cases} u_1 =1, & \\ u_{n+1} =\sqrt{2u_n^2+1}. \end{cases} \]

    Mostre, usando indução matemática, que $u_n=\sqrt{2^n-1}$, para qualquer $n\in\mathbb{N}_1$.

  13. Verifique que se $n\in\mathbb{N}$ é ímpar, então $n^2$ é também ímpar. O que pode concluir sobre $n\in\mathbb{N}$ sabendo que $n^2$ é par?

  14. Verifique que se $x, y$ são números racionais, então $x+y$, $xy$, $-x$, $x^{-1}$ (para $x\neq 0$) são também números racionais.

    Comentário

    Ou seja, $\mathbb{Q}$ é fechado para a adição e multiplicação e contem os simétricos de todos os seus elementos e os inversos de todos os seus elementos não nulos. Mostra-se assim, uma vez que também os elementos neutros $0$ e $1$ são racionais, que $\mathbb{Q}$ é um corpo. É fácil ver que também verifica as propriedades de ordem, ou seja, $\mathbb{Q}$ é um corpo ordenado.

    Solução

    Sejam $x,y\in \mathbb{Q}$, ou seja $x=\frac{p}{q}$, $y=\frac{r}{s}$, com $p,q,r,s\in \mathbb{Z}$. Então, $-x=\frac{-p}{q}$, $x^{-1}=\frac{q}{p}$, $x+y=\frac{p}{q}+\frac{r}{s}=\frac{ps+rq}{qs}$, logo $-x$, $x^{-1}$, $x+y\in\mathbb{Q}$.

  15. (Ex. I.3 de [1]) Verifique que, se $x$ é um número racional diferente de zero e $y$ um número irracional, então $x+y$, $x-y$, $xy$ e $y/x$ são irracionais; mostre também que, sendo $x$ e $y$ irracionais, a sua soma, diferença, produto e quociente podem ser ou não ser irracionais.

    Solução

    Seja $x\neq 0$ um racional e $y$ um irracional. Se $x+y$ fosse racional, uma vez que a soma e a subtracção de dois racionais é também racional, teriamos que $(x+y)-x$ seria racional. Mas $(x+y)-x =y$, logo $y$ seria racional, o que contradiz a hipótese. Conclui-se que $x+y$ é irracional.

    Para mostrar que $x-y$, $xy$ e $y/x$ são irracionais, a prova é semelhante (usando o facto da soma, divisão e multiplicação de racionais ser racional).

    Sendo $x$ e $y$ irracionais, a sua soma, diferença, produto e quociente podem ser ou não ser irracionais. Por exemplo: como $\sqrt{2}\in \mathbb{R}\setminus\mathbb{Q}$, \[\sqrt{2}+\sqrt{2}=2\sqrt{2}\in \mathbb{R}\setminus\mathbb{Q}, \quad \sqrt{2}+(-\sqrt{2})=0\in \mathbb{Q},\quad \sqrt{2}\sqrt{2}=2\in \mathbb{Q}, \dots\]

  16. Mostre que existem irracionais $x$ e $y$ tais que $x^y$ é racional.
    Sugestão

    Use $\sqrt{2}\notin\mathbb{Q}$ e considere $\sqrt{2}^{\sqrt{2}}$... Não precisa decidir se $\sqrt{2}^{\sqrt{2}}\in\mathbb{Q}$ ou $\sqrt{2}^{\sqrt{2}}\notin\mathbb{Q}$.

  17.  Inserir/Editar âncora(Ex. 1.2 de [2]) Considere os seguintes subconjuntos de $\mathbb{R}$:

    \[ A=\left\{x\in\mathbb{R}:|x|\geq \frac{x}{2}+2\right\},\qquad B=[-3,4],\qquad C=\mathbb{R}\setminus\mathbb{Q}. \]
    1. Mostre que $A\cap B =\left[-3,-\frac{4}{3}\right]\cup \{4\}$.
    2. Indique, se existirem em $\mathbb{R}$, $\sup A$, $\min(A\cap B)$, $\max(A\cap B)$, $\inf(A\cap B\cap C)$, $\sup(A\cap B\cap C)$ e $\min(A\cap B\cap C)$.
  18.  Inserir/Editar âncora(Exame de 19/1/2000) Considere os conjuntos $A$ e $B$ definidos por

    \[ A=\left\{ x\in\mathbb{R}\,:\,\frac{x-1}{x\log x}>0\right\}, \qquad B=\left\{ x\in\mathbb{R}\,:\, x=-\frac{1}{n}\,,n\in\mathbb{N}_{1}\right\}. \]

    Mostre que o conjunto $A$ é igual a $\mathbb{R}^{+}\setminus \{1\}.$ Determine, caso existam, ou justifique que não existem, o supremo, o ínfimo, o máximo e o mínimo dos conjuntos $A$ e $A\cup B$.

  19. (Ex. 1.8 de [2]) Considere os conjuntos

    \[ A=\left\{ x\in\mathbb{R}: \frac{1}{\log x}\geq 1\right\}, \qquad B=\left\{ 1-\frac{(-1)^n}{n},n\in\mathbb{N}_{1}\right\}. \]

    Para cada um dos conjuntos $A$ e $B$, indique o conjunto dos majorantes, o conjunto dos minorantes e, no caso de existirem (em $\mathbb{R}$), o supremo, o ínfimo, o máximo e o mínimo.

  20. Sejam $A$, $B$ e $C$ os seguintes subconjuntos de $\mathbb{R}$:

    \begin{gather*} A=\left\{x\in\mathbb{R}: x^{2}+2|x|\gt 3\right\},\qquad B=\left]0,\sqrt{2}\right[,\\ C=\left\{\sqrt{2}-\frac{1}{n}: n\in\mathbb{N}_{1}\right\}. \end{gather*}
    1. Calcule $A$ sob a forma de uma reunião de intervalos.
    2. Indique, caso existam, $\inf A$, $\min A\cap B$, $\max A\cap B$, $\max A\cap B\cap \mathbb{Q}$, $\inf A\cap B\cap \mathbb{Q}$, $\max C$, $\max B\setminus C$.
  21.  Inserir/Editar âncora(Exame de 2000) Sejam $A$ e $B$ os subconjuntos de $\mathbb{R}$ definidos por

    \[ A=\{x\in\mathbb{R}: |x-1|\leq x^{2}-1\},\qquad B=[-2,2]. \]
    1. Determine $A$ sob a forma de reunião de intervalos.
    2. Determine, se existirem em $\mathbb{R}$, o máximo e o mínimo de $A\cap B$ e o supremo, ínfimo, máximo e mínimo de $(A\cap B)\setminus\mathbb{Q}$.
  22. (Exame de 30/11/2002) Considere os seguintes conjuntos de $\mathbb{R}$:

    \[ A=\left\{x:|x^2-2|\leq 2x+1\right\},\qquad B=\mathbb{Q},\qquad C=\left\{\frac{1}{k^2}:k\in\mathbb{N}_1\right\}. \]
    1. Mostre que $A=\left[-1+\sqrt{2},3\right]$.
    2. Determine, se existirem, o supremo, ínfimo, máximo e mínimo de $A\cap B$ e $C$.
  23. (Exame de 16/1/2004) Considere os seguintes conjuntos de números reais:

    \[ A=\left\{x:\frac{x^{2}-1}{x}\geq |x-1|\right\},\qquad B=\left\{x:\sen x = 0\right\},\qquad C=\mathbb{Q}. \]
    1. Mostre que $A=\left[-\frac{1}{2}, 0\right[\cup \left[1, +\infty\right[$.
    2. Escreva os conjuntos dos majorantes e minorantes de $A\cap C$ e $B\cap C$. Calcule ou conclua da não existência de $\sup A$, $\inf A\cap C$, $\min A\cap C$, $\min B$, $\sup B\cap C$.
    Solução
    1. Para determinar $A$ resolvemos a inequação $\frac{x^{2}-1}{x}\geq |x-1|$ sucessivamente para $x\geq 1$ e $x\leq 1$.

      Para $x\geq 1$ temos $|x-1|=x-1$ pelo que consideramos a inequação \begin{align*}&\frac{x^{2}-1}{x}\geq x-1 \\ & \Leftrightarrow \frac{x^{2}-1}{x}- (x-1)\geq 0  \\ & \Leftrightarrow \frac{x-1}{x}\geq 0\end{align*} pelo que verificamos que todos os números reais maiores ou iguais a $1$ estão no conjunto $A$.

      Para $x\leq 1$ temos $|x-1|=-x+1$ pelo que consideramos a inequação \begin{align*}  &\frac{x^{2}-1}{x}\geq -x+1 \\ & \Leftrightarrow \frac{x^{2}-1}{x} + x -1 \geq 0  \\ & \Leftrightarrow \frac{2x^2-x-1}{x}\geq 0 \\ & \Leftrightarrow \frac{2(x-1)(x+\frac{1}{2})}{x}\geq 0 \end{align*} Por análise do sinal dos vários factores do lado esquerdo desta inequação e notando que $x=0$ não faz parte do domínio da expressão, conclui-se que as soluções para $x\leq 1$ correspondem a $x\in {\left[-\frac{1}{2}, 0\right[}$.

      Coligindo a informação obtida, obtivemos de facto que $A={\left[-\frac{1}{2}, 0\right[} \cup {[1,+\infty[}$.

    2. Notamos que $B=\{x\in\mathbb{R}:x=k\pi, k\in\mathbb{Z}\}$ e que todos os elementos de $B$ são irracionais exceptuando-se $0$. Assim \[B\cap C= \left\{0\right\}.\] O conjunto dos majorantes de $A\cap C$ é vazio e o conjunto dos minorantes de $A\cap C$ é ${\left]-\infty, -\frac{1}{2}\right]}$. O conjunto dos minorantes de $B\cap C$ é $]-\infty, 0]$ e o conjunto dos majorantes $[0, +\infty[$.

      Assim $\sup A$ não existe, $\inf (A\cap C)=-\frac{1}{2}=\min(A\cap C)$, $\min B$ não existe e $\sup (B\cap C)=0$.

  24. (Teste de 12/11/2005) Considere os seguintes conjuntos de $\mathbb{R}$:

    \[ A=\left\{x:x\geq 0\;\wedge\; \frac{x^4-4}{|x-1|}\leq 0\right\},\qquad B=\left\{x:x\geq 0\;\wedge\; \exists_{k\in\mathbb{N}} \; kx\notin\mathbb{Q}\right\}. \]
    1. Mostre que $A=\left[0,\sqrt{2}\right]\setminus\{1\}$ e justifique que $B={[0,+\infty[}\setminus{\mathbb{Q}}$.
      Solução

      Começamos por notar que \begin{align*} \frac{x^4-4}{|x-1|}\leq 0 & \; \Leftrightarrow \; \frac{(x^2-2)(x^2+2)}{|x-1|}\leq 0 \\ & \; \Leftrightarrow \; x^2-2 \leq 0 \; \wedge \; |x-1|\neq 0 \quad \text{(porque $x^2+2>0$ e $|x-1|\geq 0$)} \\ & \; \Leftrightarrow \; x \in \left[-\sqrt{2}, \sqrt{2}\right] \setminus \{1\}. \end{align*}

      Então, \[ A=\left\{x\in\mathbb{R}: x\geq 0 \; \wedge \; x \in \left[-\sqrt{2}, \sqrt{2}\right] \setminus \{1\}\right\} = \left[0, \sqrt{2}\right]\setminus \{1\}. \]

      Relativamente a $B$ começamos por notar que se existe um $k\in\mathbb{N}$ tal que $kx\not\in\mathbb{Q}$ então $x\not\in \mathbb{Q}$ pois, caso contrário, $kx\in\mathbb{Q}$ para todo o $k\in\mathbb{N}$. Portanto $B\subset\mathbb{R}\setminus \mathbb{Q}$. Reciprocamente, se $x\in \mathbb{R}\setminus \mathbb{Q}$ então $1\cdot x = x\not\in\mathbb{Q}$. Portanto $B$ é de facto o conjunto dos números irracionais positivos.

    2. Determine, ou mostre que não existem, o supremo, ínfimo, máximo e mínimo de cada um dos conjuntos $A$ e $A\setminus B$.
      Solução

      Notamos que $A\setminus B=\left(\left[0,\sqrt{2}\right[\setminus \{1\}\right)\cap \mathbb{Q}$. Então, \begin{gather*} \sup A = \sup A\setminus B = \sqrt{2} =\max A, \\ \inf A = \inf A\setminus B = 0 = \min A = \min A\setminus B. \end{gather*} $A\setminus B$ não tem máximo pois $\sup A \setminus B= \sqrt{2} \not\in\mathbb{Q}$.

  25. (Teste de 29/4/2006) Considere os seguintes subconjuntos de $\mathbb{R}$:

    \[ A = \left\{ x: \; \frac{x^2-2}{|x|-1} \leq 0 \right\}, \quad B=\left\{ 2^{n/2}: n\in\mathbb{N}_1\right\}. \]
    1. Mostre que $A={\left[-\sqrt{2},-1\right[}\cup {\left]1,\sqrt{2}\right]}$.
    2. Determine ou justifique que não existem, os supremo, máximo, ínfimo e mínimo de cada um dos conjuntos $A\cap \mathbb{Q}$, $B$ e $B\cap \mathbb{Q}$.
  26. (Ex. 1.10 de [2]) Seja $A$ um subconjunto de $\mathbb{R}$, majorado e não vazio, e seja $m$ um majorante de $A$, distinto do supremo desse conjunto. Mostre que existe $\epsilon \gt 0$ tal que $V_\epsilon(m)\cap A=\emptyset$.

  27. (Ex. I.5 de [1]) Sejam $A$ e $B$ dois subconjuntos de $\mathbb{R}$ tais que $A\subset B$ e suponha que $A$ é não vazio e $B$ é majorado. Justifique que existem os supremos de $A$ e $B$ e prove que se verifica $\sup A\leq\sup B$.

    Solução

    Se $B$ é majorado e $A\subset B$, então $A$ é majorado e qualquer majorante de $B$ é majorante de $A$ (directamente da definição de majorante). Por outro lado $A\neq \emptyset \wedge A\subset B \Rightarrow B\neq \emptyset$. Logo como $A$ e $B$ são majorados e não-vazios, o axioma do supremo garante que $\sup A$ e $\sup B$ existem. Como $\sup B$ é majorante de $B$ será também majorante de $A$, logo $\sup A\leq\sup B$.

  28. (Ex. 1.12 de [2]) Sendo $U$ e $V$ dois subconjuntos majorados e não vazios de $\mathbb{R},$ tais que $\sup U\lt \sup V,$ justifique (de forma precisa e abreviada) as afirmações seguintes:

    1. Se $x\in U,$ então $x\lt \sup V$.
    2. Existe pelo menos um $y\in V$ tal que $y\gt \sup U$.
  29. (Ex. 1.14 de [2]) Sejam $A$ e $B$ dois subconjuntos de $\mathbb{R}$.

    1. Prove que, se $\sup A \lt \inf B,$ $A$ e $B$ são disjuntos.
    2. Mostre, por meio de exemplos, que se for $\sup A\gt \inf B\;\wedge \;\sup B\gt\inf A,$ $A$ e $B$ podem ser ou não disjuntos.

Bibliografia

  1. J. Campos Ferreira. Introdução à Análise Matemática. Fundação Calouste Gulbenkian, Lisboa.
  2. Departamento de Matemática do Instituto Superior Técnico. Exercícios de Análise Matemática I/II, 2ª edição, 2005. IST Press, Lisboa.

Última edição desta versão: João Palhoto Matos, 05/09/2021 09:09:53.